LSAT and Law School Admissions Forum

Get expert LSAT preparation and law school admissions advice from PowerScore Test Preparation.

User avatar
 Dave Killoran
PowerScore Staff
  • PowerScore Staff
  • Posts: 5849
  • Joined: Mar 25, 2011
|
#27198
Complete Question Explanation
(The complete setup for this game can be found here: lsat/viewtopic.php?t=11436)

The correct answer choice is (C)

This is a challenging Global question. Both answer choice (A) and answer choice (E) can be eliminated because M and G can never be in the same group of five ranked products (see the discussion of the last rule above). Answer choices (B) and (D) have an identical structure: they each play on relationships created by combining the second rule with the fourth or fifth rule. For answer choice (B), as discussed above, when H is selected for testing then F must rank first. For answer choice (D), also as discussed above, when K is selected for testing then G must rank first. Hence, both answer choice (B) and answer choice (D) can be eliminated because each is impossible. By process of elimination, answer choice (C) is correct.
 g_lawyered
  • Posts: 211
  • Joined: Sep 14, 2020
|
#89083
Hi P.S.,
This was a difficult question for me as I narrowed down between Answer choice B and C.

Is it correct to infer that ONLY F or G could be 1st (I didn't recognize the 2 Solution Sets of having either F or G 1st only). I had some trouble understanding this rule because I understood that other variables could go 1st. For example, in answering Question #7 I tested answer choice C. I came up with Solution set in this order: I, L, K, F/H, H/F (G and M are out). Is it incorrect to say that I COULD go 1st?

I tested answer choice B and because I didn't see any rules broken, I chose it. My solution was in order of: M, L, H, F/I, I/F (G and K are out). Why is this incorrect? Can someone please explain :-?

Thanks in advance
 Adam Tyson
PowerScore Staff
  • PowerScore Staff
  • Posts: 5153
  • Joined: Apr 14, 2011
|
#89094
The rule that "Either F or G ranks first" means that those are the only variables that can be ranked first, GGIBA003@FIU.EDU. If other variables could be ranked first while those variables went elsewhere, the rule would be meaningless. Thus, every solution to the game must start with F or else with G, and never with any other variable.
 g_lawyered
  • Posts: 211
  • Joined: Sep 14, 2020
|
#89342
Thanks for your response Adam,
I have a follow up question: Is it correct to imply that any rule with EITHER and OR (such as "Either F or G ranks first") means ONLY? This means that ONLY F or G can be first. I think because I misinterpreted the rule it costed me points on this game.

Thanks in advance!
 Robert Carroll
PowerScore Staff
  • PowerScore Staff
  • Posts: 1783
  • Joined: Dec 06, 2013
|
#89601
GGIBA003,

No, it's not necessary that every "either/or" mean "only", but this is a one-to-one linear game, where five things are tested, each being ranked 1 to 5 with no ties. That means that, in the scenario, it already sets up that only one thing will be ranked 1st (and only 1 ranked 2nd, and...etc.). If F or G is first, then, to fulfill that rule, one of them has to be first. There won't be room for anything else to be first.

In short, the scenario sets up that only one thing will be first, and the rule we're talking about determines that that single thing is F or G. If one of them is first, nothing else can be, because of the scenario.

Robert Carroll
 g_lawyered
  • Posts: 211
  • Joined: Sep 14, 2020
|
#90909
That makes sense. Thanks Robert!
 concrottrox11@gmail.com
  • Posts: 29
  • Joined: Dec 07, 2021
|
#92930
I do not understand why answer choice A) is incorrect. G can rank higher than M if G is listed 1st. So why can't M and G never be in the same group of 5 ranked products?
 Adam Tyson
PowerScore Staff
  • PowerScore Staff
  • Posts: 5153
  • Joined: Apr 14, 2011
|
#92999
G cannot rank higher than M, and M cannot rank higher than G, because they can never both be ranked, concrottrox11@gmail.com. This is because of the effect of combining the last rule of the game with the first and third rules.

Rule 1 tells us that L is second.
Rule 3 says that I must be tested.
Per the last rule, If M is among the 5 that are ranked, F and H must also be ranked.

Thus, if M is ranked, the group of 5 must be L, I, M, F, and H, with G and K remaining unranked. This would also force F to be ranked first, since F and G are the only ones eligible to be in first place and G is now out of the group. The solution any time M is included can only be:

FL(H,I,M)

(meaning that F is first, L is second, and H, I, and M are all completely interchangeable in the last three spaces).

We cannot say that M is ranked higher than G in this case because G is not among those that are ranked. That would be like saying I finished a foot race ahead of Usain Bolt when Bolt wasn't participating in the race!

Likewise, if G is included, M cannot be, because we would not have room in the group to also include both F and H. So if G is ranked and M is not ranked, we could not say that G was ranked higher than M. To be ranked higher than something else, both things must be included in the rankings!

Get the most out of your LSAT Prep Plus subscription.

Analyze and track your performance with our Testing and Analytics Package.